Difference between revisions of "1962 AHSME Problems/Problem 40"

(Solution)
(Solution)
Line 5: Line 5:
  
 
==Solution==
 
==Solution==
{{solution}}
+
The series can be written as the following:
 +
 
 +
<math>\frac{1}{10} + \frac{1}{10^2} + \frac{1}{10^3} + ...</math>
 +
 
 +
<math>+ \frac{1}{10^2} + \frac{1}{10^3} + \frac{1}{10^4} + ...</math>
 +
 
 +
<math>+ \frac{1}{10^3} + \frac{1}{10^4} + \frac{1}{10^5} + ...</math>
 +
 
 +
and so on.
 +
 
 +
by using the formula for infinite geometric series <math>(\frac{a}{1-r})</math>,
 +
 
 +
We can get <math>\frac{\frac{1}{10}}{1-\frac{1}{10}}</math> <math>+</math> <math>\frac{\frac{1}{10^2}}{1-\frac{1}{10}}</math> <math>+</math> <math>\frac{\frac{1}{10^3}}{1-\frac{1}{10}}</math> <math>+</math> ...
 +
Since they all have common denominators, we get <math>\frac{(\frac{1}{10} + \frac{1}{10^2} + \frac{1}{10^3})}{\frac{9}{10}}</math>.
 +
Using the infinite series formula again, we get <math>\frac{\frac{\frac{1}{10}}{1-\frac{1}{10}}}{\frac{9}{10}}</math> <math>=</math> <math>\frac{\frac{\frac{1}{10}}{\frac{9}{10}}}{\frac{9}{10}}</math> <math>=</math> <math>\frac{\frac{1}{9}}{\frac{9}{10}}</math> <math>=</math> <math>\boxed{ (B) \frac{10}{81}}</math>

Revision as of 20:27, 19 February 2016

Problem

The limiting sum of the infinite series, $\frac{1}{10} + \frac{2}{10^2} + \frac{3}{10^3} + \dots$ whose $n$th term is $\frac{n}{10^n}$ is:

$\textbf{(A)}\ \frac{1}9\qquad\textbf{(B)}\ \frac{10}{81}\qquad\textbf{(C)}\ \frac{1}8\qquad\textbf{(D)}\ \frac{17}{72}\qquad\textbf{(E)}\ \text{larger than any finite quantity}$

Solution

The series can be written as the following:

$\frac{1}{10} + \frac{1}{10^2} + \frac{1}{10^3} + ...$

$+ \frac{1}{10^2} + \frac{1}{10^3} + \frac{1}{10^4} + ...$

$+ \frac{1}{10^3} + \frac{1}{10^4} + \frac{1}{10^5} + ...$

and so on.

by using the formula for infinite geometric series $(\frac{a}{1-r})$,

We can get $\frac{\frac{1}{10}}{1-\frac{1}{10}}$ $+$ $\frac{\frac{1}{10^2}}{1-\frac{1}{10}}$ $+$ $\frac{\frac{1}{10^3}}{1-\frac{1}{10}}$ $+$ ... Since they all have common denominators, we get $\frac{(\frac{1}{10} + \frac{1}{10^2} + \frac{1}{10^3})}{\frac{9}{10}}$. Using the infinite series formula again, we get $\frac{\frac{\frac{1}{10}}{1-\frac{1}{10}}}{\frac{9}{10}}$ $=$ $\frac{\frac{\frac{1}{10}}{\frac{9}{10}}}{\frac{9}{10}}$ $=$ $\frac{\frac{1}{9}}{\frac{9}{10}}$ $=$ $\boxed{ (B) \frac{10}{81}}$